¿Secuencia única de escalera de momento angular en mecánica cuántica? -- ¿Por qué sólo hay un

¿Cómo prueba que solo hay una secuencia de estados propios de momento angular conectados por el operador de escalera, dentro del subespacio donde el módulo al cuadrado del momento angular tiene un valor propio dado?

Para ser más precisos, dejemos j ^ ser un operador de momento angular (generalizado) definido como un operador hermitiano cuyas componentes cartesianas satisfacen las relaciones de conmutación,

[ j ^ X , j ^ y ] = i j ^ z , [ j ^ y , j ^ z ] = i j ^ X , [ j ^ z , j ^ X ] = i j ^ y .
Definimos la elevación ( j ^ + ) y bajando ( j ^ ) operadores como
j ^ + = j ^ X + i j ^ y , j ^ = j ^ X i j ^ y .
Démosle un nombre genérico, operador de escalera, a j ^ + y j ^ . Llamemos al siguiente operador como el módulo cuadrado del momento angular,
j ^ 2 = j ^ X 2 + j ^ y 2 + j ^ z 2 .

De la relación de conmutación, [ j ^ 2 , j ^ X ] = 0 , estos dos operadores tienen estados propios simultáneos, que escribimos como | X α , Y β , significa que

j ^ 2 | X α , Y β = X α | X α , Y β , j ^ z | X α , Y β = Y β | X α , Y β .
De las relaciones de conmutación, [ j ^ 2 , j ^ ± ] = 0 y [ j ^ z , j ^ ± ] = ± j ^ ± , resulta que
j ^ ± | X α , Y β >= | X α , Y β ± > C ± ( X α , Y β ) ,
dónde C ± ( X α , Y β ) es un número complejo. del hecho de que
X α , Y β | ( j ^ 2 j ^ z 2 ) | X α , Y β = X α , Y β | ( j ^ X 2 + j ^ y 2 ) | X α , Y β 0 ,
resulta que
X α Y β 2 0.
Por lo tanto, el valor propio de j ^ z tiene los límites inferior y superior determinados por el valor propio de j ^ 2 . Hasta ahora tan bueno para mí.

Todos los libros de texto que he investigado siguen esencialmente un tipo de argumento para determinar los valores de X α y Y β [1]. Este argumento se basa en la afirmación de que

  • multiplicación sucesiva de j ^ + en | X α , Y min debe resultar en | X α , Y máximo D , ..... (A1)

dónde Y min y Y máximo son el valor mínimo y máximo tomado por Y β , y D es un número complejo.

Entonces, se deduce que X j = j ( j + 1 ) 2 con j = 0 , 1 / 2 , 1 , 3 / 2 , , y Y metro = metro con metro = j , j + 1 , , j 1 , j , donde cambié los índices α y β a j y metro seguir la notación convencional. Sin embargo, no veo ninguna justificación de la afirmación (A1) citada anteriormente.

Suponer que 0 < a < y eso Y β = Y min + a es un valor propio de j ^ z . ¿Qué contradicción resulta? Aplicación de j ^ + en | X α , Y min + a simplemente revelaría otra secuencia de estados propios de j ^ z con valores propios Y min + a , Y min + a + , , Y min + a + METRO , dónde METRO es el entero tal que Y máximo < Y min + a + METRO Y máximo . Especialmente, ¿qué contradicción resulta si la igualdad se cumple en la última expresión, es decir, si existe a tal que ( Y máximo Y min a ) / es un número entero y, por lo tanto, la afirmación (A1) es falsa?

Esta pregunta,

Momento angular: prueba de valores propios enteros o semienteros

probablemente tenía la intención de hacer la misma pregunta cuando se planteó en ese libro de texto (desconocido) estudiado por el OP de esa pregunta, pero las dos respuestas allí asumen el reclamo (A1) en cuestión aquí, y el OP aceptó una de las respuestas . Por lo tanto, estoy haciendo esta nueva pregunta aquí, precisando el punto de preocupación.

Esta pregunta,

Por qué s yo ( 2 , C ) operadores de subida y bajada j ± garantizar valores propios cuantificados?

pregunta sobre un punto ligeramente diferente, es decir, la existencia de operadores de escalera con paso fraccionario en .

[1] El argumento en los libros de texto es esencialmente el mismo que en esta publicación de respuesta:

https://física.stackexchange.com/a/128918/6399

Tomé la notación en parte de esta publicación.

Para preservar la invariancia de la función de onda bajo una rotación completa (de 2 π o 4 π ), obtenemos una restricción sobre los posibles estados de momento angular como números enteros (o medios enteros).

Respuestas (2)

Operador de momento angular j es el generador de rotaciones en una función de onda. Eso es si tienes un estado | ψ y desea expresarlo en términos de coordenadas que giran alrededor de un eje norte ^ por un ángulo θ , esto viene dado por

Exp ( i j norte ^ θ ) | ψ
Ahora la invariancia de | ψ bajo rotación de 2 norte π restringe los valores propios de j ser múltiplos enteros de . Por lo tanto, agotamos todos los estados posibles mientras contamos a través de operadores de escalera.

Consideremos | ψ tal que j ^ z | ψ = ( / 2 ) | ψ . Al 2 π rotación alrededor de la z eje, Exp ( i 2 π j ^ z / ) | ψ = | ψ mi i π , y adquiere el factor de fase. Este | ψ es invariable sólo sobre 4 π rotación. Ahora supongamos | ϕ es tal que j ^ z | ϕ = ( / 3 ) | ϕ . adquiere la fase mi i 2 π / 3 al 2 π rotación, y es invariante bajo 6 π rotación. Por que asi | ϕ no existe, mientras | ψ es un estado propio válido de j ^ z ?
Eso es porque j z está obligado por j . y los valores permitidos para j son múltiplos enteros de 2 y la razón por la cual se puede ver en physics.stackexchange.com/questions/174018/…
Gracias por tu respuesta y comentario. Sin embargo, no creo que mi pregunta sea respondida. Que hace j en tu respuesta y comentario significa? ¿Es un vector de operador o es un escalador? Déjame interpretar que quieres decir que el valor propio de j ^ z está acotado por un múltiplo entero de / 2 . Puedo tomar este límite superior como / 2 . Claramente, / 3 está por debajo de este límite. (Es posible que desee decir que el límite inferior es / 2 , y / 3 está por encima de este límite). Además, no entiendo dónde específicamente en el enlace al que se refiere.

En resumen, cualquier escalera de valores propios de j ^ z , otro aparte de esos j , ( j + 1 ) , , j dónde j es entero o medio entero, no terminaría al menos en una dirección y, por lo tanto, estaría en contradicción con los límites finitos que se muestran en el texto de la pregunta.

Para ser más precisos, elija cualquier estado propio | X α , Y β , dónde Y β no se supone que sea un múltiplo entero o semientero de . Aplicando el operador de elevación de forma repetitiva sobre ella, subimos por la escalera como

j ^ + k | X α , Y β = | X α , Y β + k C + ( X α , Y β + ( k 1 ) ) C + ( X α , Y β )
para k = 1 , 2 , . Dado que existe el límite superior para el valor propio de j ^ z , debe ser un entero k 0 tal que
C + ( X α , Y β + ( k 1 ) ) 0 , k = 1 , , k , C + ( X α , Y β + k ) = 0.
Escribamos el valor propio más grande en esta escalera como
m := Y β + ( k 1 ) ,
y el estado propio correspondiente como
| v 0 := | X α , m .
Desde aquí, bajamos la escalera y veremos una condición en la que la escalera se trunca en pasos finitos.

Nosotros escribimos

(1) | v k := j ^ k | v 0 = | X α , m k C ( X α , m ( k 1 ) ) C ( X α , m ) ,
para k = 1 , 2 , . Entonces, de la relación de conmutación, [ j ^ + , j ^ ] = 2 j ^ z , es decir,
(2) j ^ + j ^ = j ^ j ^ + + 2 j ^ z ,
se sigue que, por k = 1 , 2 , ,
(3) j ^ + | v k = | v k 1 k [ 2 m ( k 1 ) ] .
Para verificar esta fórmula, podemos comprobar el k = 1 caso aplicando ambos lados de la mano de eq. ( 2 ) en | v 1 , y luego podemos usar la inducción matemática para k > 1 .

Para la escalera de m k ( k = 1 , 2 , ), que aparece en la ec. ( 1 ), para truncar en un valor finito, debe haber un número entero k 0 tal que

C ( X α , Y β ( k 1 ) ) 0 , k = 1 , , k , C ( X α , Y β k ) = 0.
Esto significa que
| v k 0 , | v k + 1 = 0.
Aplicando el lado izquierdo de la ec. ( 2 ) en | v k ,
j ^ + j ^ | v k = j ^ + | v k + 1 = 0.
Aplicando el lado derecho de la ec. ( 2 ) en | v k y usando la ec. ( 3 ),
[ j ^ j ^ + + 2 j z ] | v k = j ^ | v k 1 k [ 2 m ( k 1 ) ] + | v k 2 [ m k ] = | v k ( k + 1 ) [ 2 m k ] .
Los resultados de las dos ecuaciones anteriores deben ser iguales, y dado que k + 1 > 0 , resulta que
m = k 2 .
Por lo tanto, vemos que el mayor valor propio m en la escalera debe haber sido un múltiplo entero o medio entero de , o de lo contrario, que la escalera continuaría hacia abajo infinitamente y contradiría la existencia del límite inferior.

En consecuencia, los valores propios de j ^ z debe ser

k 2 k , k 2 ( k 1 ) , , k 2 ,
por algún entero k 0 .

Tomé este argumento del libro,

  • BC Hall, Teoría cuántica para matemáticos (Springer, 2013).

Ver Prueba del Teorema 17.4 en el mismo. ecuación (17.12) del libro corresponde a la ec. ( 3 ) aquí. [ v j en el lado derecho de la Ec. (17.12) debe decir v j 1 .]